www.vorhilfe.de
Vorhilfe

Kostenlose Kommunikationsplattform für gegenseitige Hilfestellungen.
Hallo Gast!einloggen | registrieren ]
Startseite · Forum · Wissen · Kurse · Mitglieder · Team · Impressum
Forenbaum
^ Forenbaum
Status Englisch
  Status Grammatik
  Status Lektüre
  Status Korrekturlesen
  Status Übersetzung
  Status Sonstiges (Englisch)

Gezeigt werden alle Foren bis zur Tiefe 2

Navigation
 Startseite...
 Neuerdings beta neu
 Forum...
 vorwissen...
 vorkurse...
 Werkzeuge...
 Nachhilfevermittlung beta...
 Online-Spiele beta
 Suchen
 Verein...
 Impressum
Das Projekt
Server und Internetanbindung werden durch Spenden finanziert.
Organisiert wird das Projekt von unserem Koordinatorenteam.
Hunderte Mitglieder helfen ehrenamtlich in unseren moderierten Foren.
Anbieter der Seite ist der gemeinnützige Verein "Vorhilfe.de e.V.".
Partnerseiten
Weitere Fächer:

Open Source FunktionenplotterFunkyPlot: Kostenloser und quelloffener Funktionenplotter für Linux und andere Betriebssysteme
Forum "Wahrscheinlichkeitstheorie" - Bedingte Erwartungswerte
Bedingte Erwartungswerte < Wahrscheinlichkeitstheorie < Stochastik < Hochschule < Mathe < Vorhilfe
Ansicht: [ geschachtelt ] | ^ Forum "Wahrscheinlichkeitstheorie"  | ^^ Alle Foren  | ^ Forenbaum  | Materialien

Bedingte Erwartungswerte: Frage (beantwortet)
Status: (Frage) beantwortet Status 
Datum: 22:30 Sa 23.01.2010
Autor: steppenhahn

Aufgabe
X,Y seien stochastisch unabhängige Zufallsvariablen mit $E(|X|), E(|Y|) < [mm] \infty$. [/mm] Berechne:

a) E(X+Y|Y)
b) E(XY|Y) für E(|XY|) < [mm] \infty [/mm]
c) [mm] E((X+Y)^{2}|Y) [/mm] für [mm] E(Y^{2}), E(X^{2}) [/mm] < [mm] \infty. [/mm]

Hallo!

Bei den obigen Aufgaben bin ich mir nicht ganz sicher, was ich machen darf und was nicht. Ich versuche es mal im stetigen Fall:

a)

Es ist ja $E(X|Y) := [mm] \int_{\IR}x*f_{X|Y=y}(x) [/mm] dx$. Also müsste ich schreiben können:

$E(X+Y|Y) = [mm] \int_{\IR}(x+y)*f_{X|Y=y}(x) [/mm] dx = [mm] \int_{\IR}x*f_{X|Y=y}(x) [/mm] dx +  [mm] \int_{\IR}y*f_{X|Y=y}(x) [/mm] dx = E(X|Y)+E(Y|Y) = E(X) + Y$ ?,

weil E(X|Y) = E(X) wenn X,Y stochastisch unabhängig.
Irgendwie nehme ich mir das aber selbst nicht ab...

b)

Wir hatten eine Formel bewiesen: $E(X*h(Y)|Y) = h(Y)*E(X|Y)$.
Wenn ich das hier anwende: E(X*Y|Y) = Y*E(X|Y) = Y*E(X). Aber wieso muss dann nach Voraussetzung E(|XY|) < [mm] \infty [/mm] gelten?

c)

[mm] $E((X+Y)^{2}|Y) [/mm] = [mm] E(X^{2}+2*X*Y+Y^{2}|Y) [/mm] = [mm] E(X^{2}|Y)+E(2*X*Y|Y)+E(Y^{2}|Y) [/mm] = [mm] E(X^{2}) [/mm] + 2*Y*E(X) + [mm] Y^{2}$. [/mm]

Stimmt das?

Danke für Eure Hilfe!
Grüße,
Stefan

        
Bezug
Bedingte Erwartungswerte: Antwort
Status: (Antwort) fertig Status 
Datum: 07:27 So 24.01.2010
Autor: felixf

Moin Stefan!

> X,Y seien stochastisch unabhängige Zufallsvariablen mit
> [mm]E(|X|), E(|Y|) < \infty[/mm]. Berechne:
>  
> a) E(X+Y|Y)
>  b) E(XY|Y) für E(|XY|) < [mm]\infty[/mm]
>  c) [mm]E((X+Y)^{2}|Y)[/mm] für [mm]E(Y^{2}), E(X^{2})[/mm] < [mm]\infty.[/mm]
>  Hallo!
>  
> Bei den obigen Aufgaben bin ich mir nicht ganz sicher, was
> ich machen darf und was nicht. Ich versuche es mal im
> stetigen Fall:
>  
> a)
>  
> Es ist ja [mm]E(X|Y) := \int_{\IR}x*f_{X|Y=y}(x) dx[/mm]. Also
> müsste ich schreiben können:
>  
> [mm]E(X+Y|Y) = \int_{\IR}(x+y)*f_{X|Y=y}(x) dx = \int_{\IR}x*f_{X|Y=y}(x) dx + \int_{\IR}y*f_{X|Y=y}(x) dx = E(X|Y)+E(Y|Y) = E(X) + Y[/mm]
> ?,
>  
> weil E(X|Y) = E(X) wenn X,Y stochastisch unabhängig.
>  Irgendwie nehme ich mir das aber selbst nicht ab...

Ich nehme mal an, ihr habt gezeigt, dass $E(X|Y) = E(X)$ und $E(Y|Y) = Y$ ist?

In dem Fall liegt das Problem beim Integral.

Einmal musst du evtl. zeigen, dass [mm] $f_{X|Y=y}$ [/mm] ueberhaupt existiert. Und dann ist da noch die Frage: was ist $y$? So macht das ganze zumindest nicht so viel Sinn.

Habt ihr evtl. in der VL gezeigt, dass [mm] $E(\bullet|Y)$ [/mm] linear ist? Dann brauchst du keine Integrale zu verwenden.

> b)
>  
> Wir hatten eine Formel bewiesen: [mm]E(X*h(Y)|Y) = h(Y)*E(X|Y)[/mm].
>  
> Wenn ich das hier anwende: E(X*Y|Y) = Y*E(X|Y) = Y*E(X).
> Aber wieso muss dann nach Voraussetzung E(|XY|) < [mm]\infty[/mm]
> gelten?

Was hat die Formel denn fuer Voraussetzungen?

> c)
>  
> [mm]E((X+Y)^{2}|Y) = E(X^{2}+2*X*Y+Y^{2}|Y) = E(X^{2}|Y)+E(2*X*Y|Y)+E(Y^{2}|Y) = E(X^{2}) + 2*Y*E(X) + Y^{2}[/mm].
>  
> Stimmt das?

Warum gilt $E(|XY|) < [mm] \infty$? [/mm] (Passenden Satz verwenden!) Das brauchst du damit du b) benutzen kannst.

Ansonsten ist's richtig.

LG Felix


Bezug
                
Bezug
Bedingte Erwartungswerte: Frage (beantwortet)
Status: (Frage) beantwortet Status 
Datum: 11:22 So 24.01.2010
Autor: steppenhahn

Hallo Felix,

danke für deine Antwort!

> Ich nehme mal an, ihr habt gezeigt, dass [mm]E(X|Y) = E(X)[/mm] und
> [mm]E(Y|Y) = Y[/mm] ist?

Ja, wir haben: $E(X*h(Y)|Y) = h(Y)*E(X|Y)$ für jede messbare Funktion h. Und wenn ich X = 1 konstant wähle, dann ist die ja eigentlich unabhängig von Y, und dann steht da: E(h(Y)|Y) = h(Y)*E(1|Y) = h(Y). Weil wir haben auch gezeigt, dass E(X|Y) = E(X), wenn X,Y stochastisch unabhängig.

> Habt ihr evtl. in der VL gezeigt, dass [mm]E(\bullet|Y)[/mm] linear
> ist? Dann brauchst du keine Integrale zu verwenden.

Es steht im Skript, aber mit "Nachrechnen". Könnte ich das mal versuchen?


> > b)
>  >  
> > Wir hatten eine Formel bewiesen: [mm]E(X*h(Y)|Y) = h(Y)*E(X|Y)[/mm].
>  
> >  

> > Wenn ich das hier anwende: E(X*Y|Y) = Y*E(X|Y) = Y*E(X).
> > Aber wieso muss dann nach Voraussetzung E(|XY|) < [mm]\infty[/mm]
> > gelten?
>  
> Was hat die Formel denn fuer Voraussetzungen?

Die Formel hat eigentlich nur die Voraussetzung, dass h messbar ist, das verwirrt mich auch so...

> > c)
>  >  
> > [mm]E((X+Y)^{2}|Y) = E(X^{2}+2*X*Y+Y^{2}|Y) = E(X^{2}|Y)+E(2*X*Y|Y)+E(Y^{2}|Y) = E(X^{2}) + 2*Y*E(X) + Y^{2}[/mm].
> >  

> > Stimmt das?
>  
> Warum gilt [mm]E(|XY|) < \infty[/mm]? (Passenden Satz verwenden!)
> Das brauchst du damit du b) benutzen kannst.

Mhh...
Ich weiß, dass X und Y stochastisch unabhängig. Dann müsste gelten: $E(|X*Y|) = E(|X|)*E(|Y|)$. Und nun weiß ich, dass [mm] E(X^{2}),E(Y^{2}) [/mm] < [mm] \infty [/mm] sind. Daraus bekomme ich, dass [mm] E(X^{2})*E(Y^{2}) [/mm] = [mm] E((X*Y)^{2}) [/mm] < [mm] \infty... [/mm]
Aber wie kann ich das benutzen, um E(|X*Y|) < [mm] \infty [/mm] zu beweisen?

Vielen Dank für Eure Hilfe!!!
Grüße,
Stefan

Bezug
                        
Bezug
Bedingte Erwartungswerte: Antwort
Status: (Antwort) fertig Status 
Datum: 19:44 So 24.01.2010
Autor: felixf

Hallo Stefan!

> > Ich nehme mal an, ihr habt gezeigt, dass [mm]E(X|Y) = E(X)[/mm] und
> > [mm]E(Y|Y) = Y[/mm] ist?
>  
> Ja, wir haben: [mm]E(X*h(Y)|Y) = h(Y)*E(X|Y)[/mm] für jede messbare
> Funktion h. Und wenn ich X = 1 konstant wähle, dann ist
> die ja eigentlich unabhängig von Y, und dann steht da:
> E(h(Y)|Y) = h(Y)*E(1|Y) = h(Y). Weil wir haben auch
> gezeigt, dass E(X|Y) = E(X), wenn X,Y stochastisch
> unabhängig.

Ok :)

> > Habt ihr evtl. in der VL gezeigt, dass [mm]E(\bullet|Y)[/mm] linear
> > ist? Dann brauchst du keine Integrale zu verwenden.
>  
> Es steht im Skript, aber mit "Nachrechnen". Könnte ich das
> mal versuchen?

Das folgt direkt aus der Integralschreibweise bzw. Definition von [mm] $E(\bullet|Y)$ [/mm] als Ideal. Du brauchst dazu "nur" die vollstaendige Definition von der Zufallsvariablen [mm] $E(\bullet|Y)$. [/mm]

> > > b)
>  >  >  
> > > Wir hatten eine Formel bewiesen: [mm]E(X*h(Y)|Y) = h(Y)*E(X|Y)[/mm].
> >  

> > >  

> > > Wenn ich das hier anwende: E(X*Y|Y) = Y*E(X|Y) = Y*E(X).
> > > Aber wieso muss dann nach Voraussetzung E(|XY|) < [mm]\infty[/mm]
> > > gelten?
>  >  
> > Was hat die Formel denn fuer Voraussetzungen?
>  
> Die Formel hat eigentlich nur die Voraussetzung, dass h
> messbar ist, das verwirrt mich auch so...

Eventuell reicht das auch.

Ich vermute mittlerweile, die Voraussetzung $E(|Z|) < [mm] \infty$ [/mm] wird dafuer benoetigt, um $E(Z|Y)$ zu definieren. Guck evtl. mal im Skript wie $E(Z|Y)$ genau definiert wird, insb. was vorausgesetzt wird.

> > > c)
>  >  >  
> > > [mm]E((X+Y)^{2}|Y) = E(X^{2}+2*X*Y+Y^{2}|Y) = E(X^{2}|Y)+E(2*X*Y|Y)+E(Y^{2}|Y) = E(X^{2}) + 2*Y*E(X) + Y^{2}[/mm].
>  
> > >  

> > > Stimmt das?
>  >  
> > Warum gilt [mm]E(|XY|) < \infty[/mm]? (Passenden Satz verwenden!)
> > Das brauchst du damit du b) benutzen kannst.
>  
> Mhh...
>  Ich weiß, dass X und Y stochastisch unabhängig. Dann
> müsste gelten: [mm]E(|X*Y|) = E(|X|)*E(|Y|)[/mm]. Und nun weiß
> ich, dass [mm]E(X^{2}),E(Y^{2})[/mm] < [mm]\infty[/mm] sind. Daraus bekomme
> ich, dass [mm]E(X^{2})*E(Y^{2})[/mm] = [mm]E((X*Y)^{2})[/mm] < [mm]\infty...[/mm]
>  Aber wie kann ich das benutzen, um E(|X*Y|) < [mm]\infty[/mm] zu
> beweisen?

Hattet ihr zufaellig die hoeldersche Ungleichung fuer Integrale? Das ist genau das, was du hier brauchst.

LG Felix


Bezug
                                
Bezug
Bedingte Erwartungswerte: Frage (überfällig)
Status: (Frage) überfällig Status 
Datum: 23:35 Di 26.01.2010
Autor: steppenhahn

Hallo Felix,

danke für deine Antwort!

> > > Habt ihr evtl. in der VL gezeigt, dass [mm]E(\bullet|Y)[/mm] linear
> > > ist? Dann brauchst du keine Integrale zu verwenden.
>  >  
> > Es steht im Skript, aber mit "Nachrechnen". Könnte ich das
> > mal versuchen?
>  
> Das folgt direkt aus der Integralschreibweise bzw.
> Definition von [mm]E(\bullet|Y)[/mm] als Ideal. Du brauchst dazu
> "nur" die vollstaendige Definition von der Zufallsvariablen
> [mm]E(\bullet|Y)[/mm].

Mir ist es dennoch nicht klar. Ich weiß, dass

[mm] $E(X|Y=y):=\int_{\IR}y*f_{X|Y=y}(x) [/mm] dx$,

wobei [mm] $f_{X|Y=y}(x) [/mm] := [mm] \frac{f_{X,Y}(x,y)}{f_{Y}(y)}$ [/mm] (und "manchmal" 0, falls [mm] f_{Y}(y) [/mm] = 0).
Aber was ist jetzt E(a*X+b*Y|Z) ? Ich scheitere schon an der Integral-Schreibweise, weil theoretisch muesste meine Funktion f doch jetzt von drei Parametern abhängen? [mm] f_{X,Y,Z} [/mm] ?

>  >  Ich weiß, dass X und Y stochastisch unabhängig. Dann
> > müsste gelten: [mm]E(|X*Y|) = E(|X|)*E(|Y|)[/mm]. Und nun weiß
> > ich, dass [mm]E(X^{2}),E(Y^{2})[/mm] < [mm]\infty[/mm] sind. Daraus bekomme
> > ich, dass [mm]E(X^{2})*E(Y^{2})[/mm] = [mm]E((X*Y)^{2})[/mm] < [mm]\infty...[/mm]
>  >  Aber wie kann ich das benutzen, um E(|X*Y|) < [mm]\infty[/mm] zu
> > beweisen?
>  
> Hattet ihr zufaellig die hoeldersche Ungleichung fuer
> Integrale? Das ist genau das, was du hier brauchst.

Reicht theoretisch auch die Cauchy-Schwarz-Ungleichung? (Weil von Hoelder hab ich zwar schon gehoert, aber ich weiß nicht, ob wir sie benutzen dürfen.).
Ich habe es jetzt so geschrieben:

[mm] \infty [/mm] > [mm] E(X^{2})*E(Y^{2}) [/mm] = [mm] \left(\int_{\IR}x^{2}*f_{X}(x)dx\right)*\left(\int_{\IR}y^{2}*f_{Y}(y) dy\right) \ge [/mm] ... [mm] \ge [/mm] = [mm] \left(\int_{\IR}|x|*f_{X}(x) dx\right)*\left(\int_{\IR}|y|*f_{Y}(y) dy\right) [/mm] = E(|X|)*E(|Y|) = E(|X*Y|).

Aber wie genau funktioniert das jetzt in der Mitte :-) ?

Danke für Eure Hilfe!
Grüße,
Stefan

Bezug
                                        
Bezug
Bedingte Erwartungswerte: Antwort
Status: (Antwort) fertig Status 
Datum: 01:09 Fr 29.01.2010
Autor: felixf

Moin Stefan!

Sorry das ich erst so spaet schreibe, irgendwie hab ich's immer verpennt...

> > > > Habt ihr evtl. in der VL gezeigt, dass [mm]E(\bullet|Y)[/mm] linear
> > > > ist? Dann brauchst du keine Integrale zu verwenden.
>  >  >  
> > > Es steht im Skript, aber mit "Nachrechnen". Könnte ich das
> > > mal versuchen?
>  >  
> > Das folgt direkt aus der Integralschreibweise bzw.
> > Definition von [mm]E(\bullet|Y)[/mm] als Ideal. Du brauchst dazu
> > "nur" die vollstaendige Definition von der Zufallsvariablen
> > [mm]E(\bullet|Y)[/mm].
>  
> Mir ist es dennoch nicht klar. Ich weiß, dass
>  
> [mm]E(X|Y=y):=\int_{\IR}y*f_{X|Y=y}(x) dx[/mm],
>  
> wobei [mm]f_{X|Y=y}(x) := \frac{f_{X,Y}(x,y)}{f_{Y}(y)}[/mm] (und
> "manchmal" 0, falls [mm]f_{Y}(y)[/mm] = 0).

Naja, das ist $E(X|Y=y)$, aber nicht $E(X|Y)$.

Du solltest auch besser mit einer anderen Formel fuer den Erwartungswert arbeiten (also aehnlich zu dem was ich gleich schreibe, halt mit der bedingten Erwartung), naemlich [mm] $\int_\Omega X(\omega) d\mathbb{P}(\omega)$, [/mm] und nicht mit [mm] $\int_\IR [/mm] x [mm] f_X(x) [/mm] dx$. Mit dem ersten bekommst du naemlich $E(X + Y) = E(X) + E(Y)$ sofort hin, mit dem zweiten nicht.

>  Aber was ist jetzt E(a*X+b*Y|Z) ? Ich scheitere schon an
> der Integral-Schreibweise, weil theoretisch muesste meine
> Funktion f doch jetzt von drei Parametern abhängen?
> [mm]f_{X,Y,Z}[/mm] ?

Genau... Deswegen arbeite lieber mit ner anderen Darstellung des Erwartungswertes.

> >  >  Ich weiß, dass X und Y stochastisch unabhängig. Dann

> > > müsste gelten: [mm]E(|X*Y|) = E(|X|)*E(|Y|)[/mm]. Und nun weiß
> > > ich, dass [mm]E(X^{2}),E(Y^{2})[/mm] < [mm]\infty[/mm] sind. Daraus bekomme
> > > ich, dass [mm]E(X^{2})*E(Y^{2})[/mm] = [mm]E((X*Y)^{2})[/mm] < [mm]\infty...[/mm]
>  >  >  Aber wie kann ich das benutzen, um E(|X*Y|) < [mm]\infty[/mm]
> zu
> > > beweisen?
>  >  
> > Hattet ihr zufaellig die hoeldersche Ungleichung fuer
> > Integrale? Das ist genau das, was du hier brauchst.
>  
> Reicht theoretisch auch die Cauchy-Schwarz-Ungleichung?

Stimmt, die reicht voellig ;-)

> (Weil von Hoelder hab ich zwar schon gehoert, aber ich
> weiß nicht, ob wir sie benutzen dürfen.).
>  Ich habe es jetzt so geschrieben:
>  
> [mm]\infty[/mm] > [mm]E(X^{2})*E(Y^{2})[/mm] =
> [mm]\left(\int_{\IR}x^{2}*f_{X}(x)dx\right)*\left(\int_{\IR}y^{2}*f_{Y}(y) dy\right) \ge[/mm]
> ... [mm]\ge[/mm] = [mm]\left(\int_{\IR}|x|*f_{X}(x) dx\right)*\left(\int_{\IR}|y|*f_{Y}(y) dy\right)[/mm]
> = E(|X|)*E(|Y|) = E(|X*Y|).

Nein, so geht das nicht. Aber so: [mm] $\infty [/mm] > [mm] \sqrt{E(X^2) E(Y^2)} [/mm] = [mm] \sqrt{\int_\Omega X(\omega)^2 d\IP(\omega) \cdot \int_\Omega Y(\omega)^2 d\IP(\omega)} \underset{\text{Cauchy-Schwarz}}{\ge} \int_\Omega |X(\omega) Y(\omega)| d\IP(\omega) [/mm] = E(|X Y|)$.

LG Felix


Bezug
                                                
Bezug
Bedingte Erwartungswerte: Mitteilung
Status: (Mitteilung) Reaktion unnötig Status 
Datum: 22:32 Fr 29.01.2010
Autor: steppenhahn

Hallo Felix,

> Sorry das ich erst so spaet schreibe, irgendwie hab ich's
> immer verpennt...

Du weißt selbst, dass das kein Problem ist - schließlich hilfst du mir!
:-)
Dafür danke ich dir!

> > [mm]E(X|Y=y):=\int_{\IR}y*f_{X|Y=y}(x) dx[/mm],
>  >  
> > wobei [mm]f_{X|Y=y}(x) := \frac{f_{X,Y}(x,y)}{f_{Y}(y)}[/mm] (und
> > "manchmal" 0, falls [mm]f_{Y}(y)[/mm] = 0).
>  
> Naja, das ist [mm]E(X|Y=y)[/mm], aber nicht [mm]E(X|Y)[/mm].
>  
> Du solltest auch besser mit einer anderen Formel fuer den
> Erwartungswert arbeiten (also aehnlich zu dem was ich
> gleich schreibe, halt mit der bedingten Erwartung),
> naemlich [mm]\int_\Omega X(\omega) d\mathbb{P}(\omega)[/mm], und
> nicht mit [mm]\int_\IR x f_X(x) dx[/mm]. Mit dem ersten bekommst du
> naemlich [mm]E(X + Y) = E(X) + E(Y)[/mm] sofort hin, mit dem zweiten
> nicht.

Ja... Ich habe es trotzdem mit der obigen Variante jetzt hinbekommen. Das Problem ist, dass unsere Vorlesung eine Einführungsvorlesung ist, wir kennen "Integrieren nach [mm] \IP [/mm] " nicht.

>  >  Ich habe es jetzt so geschrieben:
>  >  
> > [mm]\infty[/mm] > [mm]E(X^{2})*E(Y^{2})[/mm] =
> >
> [mm]\left(\int_{\IR}x^{2}*f_{X}(x)dx\right)*\left(\int_{\IR}y^{2}*f_{Y}(y) dy\right) \ge[/mm]
> > ... [mm]\ge[/mm] = [mm]\left(\int_{\IR}|x|*f_{X}(x) dx\right)*\left(\int_{\IR}|y|*f_{Y}(y) dy\right)[/mm]
> > = E(|X|)*E(|Y|) = E(|X*Y|).
>  
> Nein, so geht das nicht. Aber so: [mm]\infty > \sqrt{E(X^2) E(Y^2)} = \sqrt{\int_\Omega X(\omega)^2 d\IP(\omega) \cdot \int_\Omega Y(\omega)^2 d\IP(\omega)} \underset{\text{Cauchy-Schwarz}}{\ge} \int_\Omega |X(\omega) Y(\omega)| d\IP(\omega) = E(|X Y|)[/mm].

Danke, das werd' ich mir dann für die nächste Vorlesung merken :-)
Ich habe nämlich gerade entdeckt, dass in den Voraussetzungen der Aufgabe noch stand, dass E(|X|) und E(|Y|) beide endlich sind - damit wird das Problem natürlich trivial.

Grüße,
Stefan

Bezug
Ansicht: [ geschachtelt ] | ^ Forum "Wahrscheinlichkeitstheorie"  | ^^ Alle Foren  | ^ Forenbaum  | Materialien


^ Seitenanfang ^
www.englischraum.de
[ Startseite | Forum | Wissen | Kurse | Mitglieder | Team | Impressum ]